The Relativity of Simultaneity: A Fundamental Concept in Special Relativity

In summary, RoS is a consequence of time dilation, which is a feature of the Lorentz transform. It is not a separate stand-alone component of SR.
  • #211
mangaroosh said:
Neither, they are not illusions, they are entirely real but in relativity they arise from geometry as you mention. The length of an object is the difference between the spatial coordinates of its endpoints and that difference will vary if you rotate the axes.
...
There is no physical effect, in its rest frame the object is unchanged so no cause is required. In relativity, "contraction" is the difference between the extent measured using two different coordinate schemes and the cause of that is the angle between the axes.
...
There are no physical effects in the rest frame to be caused, just geometry.

This is an issue I have real difficulty with.

Many people do but this forum exists precisely for that reason. I've tried to address the other points in your post quickly so we can get them out of the way, this aspect is where we need to put the real effort if you want to break the logjam and start understanding SR. I've got to do other stuff now but I'll try to get back to this later or maybe tomorrow. I want to do some work that will help as it comes up often but it will take some time.
 
Physics news on Phys.org
  • #212
mangaroosh said:
We seem to be talking at cross purposes here, probably because of the different interpretations associated with the term, RoS.
There are different views about the hidden reality that may explain such things as relativity of simultaneity, time dilation etc. However, the definitions of SR are purely operational definitions, as indicated in Einstein's 1905 paper.
It appears as though reference is being made to an RoS in the Lorentz transform, which I am referring to as the relativity of time co-ordinates (RoTC RoS), for the sake of discussion; and for the sake of distinguishing between the LET interpretation and the Einsteinian. There also appears to be an RoS which refers to the simultaneity of physical events, which is distinct from, but related to, the RoS of the Lorentz transform.
I'm afraid that here it shows that you have not performed enough Lorentz transformations - if at all! The Lorentz transformations describe how physical events are mapped from one co-ordinate system to the other. Thus, there is no difference between these things; moreover, introducing a different term with the same meaning cannot help for the sake of distinguishing interpretations.
When I say RoS of physical events, I mean that two events which are simultaneous in one rerference frame are not simultaneous in another. This is contrasted with absolute simultaneity where two events which are simultaneous in one reference frame, are simultaneous across all reference frames.
This notion of RoS is incompatible with absolute simultaneity, because absolute simultaneity doesn't allow for the conditions which constitute RoS i.e. if events, absolutely, happened simultaneously ("at the same time") for all observers, then it isn't possible that they weren't simultaneous for specific, idividual observers.
That may be the cause: try again with substituting velocity (you said that you understood this, but evidently you did not!). You will then realize that you are prey to the exact same confusion as is possible between "absolute velocity" and "velocity is absolute". Similarly, "absolute simultaneity" has nothing to do with "simultaneity is absolute", contrary to what you say here above.
From the discussions I have had, on here and elsewhere, together wth pretty much all of the information I have encountered on Einsteinian relativity, it appears as though Einsteinian relativity incorporates the notion of RoS of physical events (as clarified above), while Lorentzian relativity incorporates absolute simultaneity of physical events.
That misleading picture has been corrected by all the discussions that you had with people in this thread - right?
[... misunderstandings that hopefully now are gone]
[..] is there an Einsteinian interpretation of RoS that is entirely disimilar to what I refer to as (RoTC) RoS but which doesn't result in the RoS of physical events, and so would make it compatible with absolute simultaneity - as outlined above?
I repeat, for quadruple clarity: the RoS of physical events is compatible with absolute simultaneity, just as the relativity of physical velocities is compatible absolute velocity.
[..] Is RoS a consequence of the Lorentz transform,
Sure - although historically RoS was first.
or is RoS the Einsteinian interpretation of the relativity of time co-ordinates, which are a consequence of the Lorentz transform?
RoS is not an interpretation, it follows directly from a well defined operational definition of simultaneity, based on position coordinates and clocks! You can read it here:
http://www.fourmilab.ch/etexts/einstein/specrel/www/
Did Lorentz refer to the RoTC as RoS?
He referred to it as "local time" already before Einstein came around and I don't think that he adapted his jargon to that of Einstein. Moreover, he called "absolute time" "true time" or "universal time". You can read it here:
http://en.wikisource.org/wiki/The_Einstein_Theory_of_Relativity
[..] it doesn't mean that the events weren't simultaneous for both of us; it just means that our clocks tick at different rates due to the mechanics of the clock. [..]
:smile: don't worry, I understand the above; the point was why do relatively moving clocks indicate different times? The thought experiments usually highlight the fact that one clock ticks more slowly than the other i.e. they tick at different rates.
I do worry, and with ample reason: you keep confounding clock rate with relativity of simultaneity... so again, no: The indication of different times in Einstein's train illustration is not due to different clock rates.
[..] What are the interpretations of RoS that is compatible with absolute simultaneity (and presentism) that isn't, essentially, the same as the RoTC outlined above?
Would you, by any chance, know where I could read up on them?
I now gave you a link to a presentation by Lorentz here above.
and from a quick search of recent discussions:
https://www.physicsforums.com/showthread.php?p=3558212
 
  • #213
mangaroosh said:
This is an issue I have real difficulty with. [..]
You may: according to Lorentz they are physical effects and also according to Einstein these effects have physical meaning. Here we are already touching on philosophy. You could start a topic on it (or search for an open thread on that topic) :tongue2:
 
  • #214
mangaroosh said:
OK, this helps to clarify the issue I think. [..]
i hope so, and perhaps the clearest answer is to simply correct a wrong sentence.
For example, not:

"RoTC can be interpreted in two (or maybe more) ways; it can be interpreted in such a way that incorporates absolute simultaneity, or it can be interpreted in such a way that RoS is incorporated."

But:

"The existence of RoS can be interpreted in two (or maybe more) ways; it can be interpreted independently of absolute simultaneity, or it can be interpreted in such a way that only RoS exists."

And now they are understood to be compatible. :smile:

Compare:

"Relative velocity can be interpreted in two (or maybe more) ways; it can be interpreted independently of (or even due to) absolute velocity, or it can be interpreted in such a way that only relative velocities exist."

PS. what may be confusing is that words that had multiple but compatible meanings before relativity, acquired incompatible meanings from then on (e.g. "absolute"). As a result, sometimes people use the same words or phrases to mean something different, and at other times people say something very different to mean the same.
 
Last edited:
  • #215
GeorgeDishman said:
That's easy, weld three mutally perpendicular rulers to the object. Something is "at rest relative to the object" or "in the object's rest frame" if its location measured by those rulers does not change over some finite period of time.
To avoid derailing this thread, it might be worth posting in the "at rest" thread.

But just to address the point raised: a consequence of the PoR says that observers can't determine if they are in motion or at rest; in the instance stated above, observers could determine easily if they are at rest relative to those rulers, or to a specific object; relative to what, though, can they not determine their motion?


GeorgeDishman said:
Yes, and you will also need to have a grasp of "general covariance" and the effect of diffeomorphisms which is why I suggest keeping it separate.
Those concepts aren't necessarily essential to understanding that the PoR can be extended to accelerating reference frames though, would they?
 
  • #216
GeorgeDishman said:
You can but you then have a number of ad hoc and inexplicable effects conspiring to create one illusion. If you use the geometrical approach, you can also predict for example that mass will appear to increase with speed which can then be confirmed by experiment, another bizarre ad hoc effect in LET which again has no explanation. (In fact it is worse because mass no longer has the same value in different directions in LET, it is no longer a scalar quantity but becomes a tensor IIRC).

The biggest argument against taking that approach though is that it cannot be extended. By noting that energy (mass etc.) causes curvature of the geometry, Einstein explained gravity, and in particular gets the right result for a raft of observations where Newton's Law fails. If you take GR and let the masses become negligible, the result is the geometric model of SR.

It's your choice of course but if you prefer not to learn how the geometric model works, you can never move beyond the physics of pre-1917. YMMV but I think that benefit is worth the effort.

I am certainly interested in learning how it all works, but on the way to developing that understanding there are certain things which provide stumbling blocks, usually linked to a more intuitive understanding. Of course, an intuitive understanding isn't necessarily correct, but it usually takes reasoning to overcome it.

For example, how is it that we determine this "one illusion", is it not through measuring the physical effects?

The idea that physical effects are not explained in terms of a physical cause is particularly jarring; for example what is the explanation for the increase of mass with speed, under Einsteinian relativity; and what is the explanation for the physical contractions?

Also, the, perhaps, most mainstream alternative explanation i.e. the spacetime explanation (or block universe), seems to require other assumptions also, with regard to time in general, specifically past and future, which don't seem to correspond to empirical experience.

There is also the assumption of reciprocal contractions, for which there doesn't appear to be any observational evidence, that I am aware of.
 
  • #217
GeorgeDishman said:
I'll give separate replies to different points as the posts are getting too long.



When the signal is transmitted, it is in orbit around Saturn. Maxwells' Equations allow you to calculate the speed of the emitted signal and give the value 'c'. You haven't tried to actually use the equations so I guess you don't understand the consequences. What you are suggesting is that the speed of the waves would be 'c' relative to the Earth so for example when the radio signal from Cassini was passing Jupiter, its speed relative to that planet would vary by up to 30km/s depending on where Earth was in its orbit.

The same would be true for gamma rays from a GRB 10 billion light years away. You are suggesting they would have to vary their speed when passing a galaxy 8 billion light years away depending on where Earth was in its orbit, which is somewhat nonsensical since Earth didn't exist 8 billion years ago.

What you need to understand is that, other than transforms which specifically relate values in one frame to the equivalents in another, all physical equations link measurable values in one frame. If you put in numbers measured in the Cassini craft frame, you get out numbers in that same frame and can then transform those to the Earth frame (or Jupiter or any other) using the Lorentz transforms (ignoring gravity of course, we are talking SR).

Sorry, meant to mention that I'll come back to this point.
 
  • #218
mangaroosh said:
When I say RoS of physical events, I mean that two events which are simultaneous in one rerference frame are not simultaneous in another. This is contrasted with absolute simultaneity where two events which are simultaneous in one reference frame, are simultaneous across all reference frames. This notion of RoS is incompatible with absolute simultaneity, because absolute simultaneity doesn't allow for the conditions which constitute RoS i.e.for all observers, then it isn't possible that they weren't simultaneous for specific, idividual observers.

Hopefully the distinction between (RoTC) RoS and RoS of physical evetns demonstrates that they are slighty different issues; such that RoS of physical events is incompatible with absolute simultaneity.

You seem to be missing the point:As Einstein so clearly pointed out ,unless two events occur in close proximity they cannot be determined or considered to be absolutely or actually simultaneous by any observers in any frame ,of events spatially separated.
Whether or not the universe occurs everywhere as a single simultaneous event and any pair of events is either actually simultaneous [in the sense that we normally think] or not, is moot. There is simply no way to tell and LET as well as SR are both aware of and operate on this basis. To my recollection the only reference to actual simultaneity in LET is the concept that only in a frame actually at rest could clocks be actually synchronized/simultaneous. While this idea is certainly logical and could quite possibly be correct it is absolutely useless as there is, once again, no possible means of determining the actual state of motion of any system.
SO there is no possible alternative to using the purely operative definition : occurring at the same coordinate time as indicated by local clocks. With no implication or assumption of actually or absoluteness in any case whatsoever.
It becomes a question of clock synchronization.
How do you actually synchronize clocks that are at disparate locations? Obviously somewhat a catch 22 as we know it is impossible to determine actual simultaneity for any separated events including setting clocks to the same time. Well Einstein did come up with a rational method that, while it didn't produce actual synchronization, did make physics work just fine in all frames.
mangaroosh said:
the point was why do relatively moving clocks indicate different times? The thought experiments usually highlight the fact that one clock ticks more slowly than the other i.e. they tick at different rates.

Clock rates have nothing to do with it. If two systems have clocks that are actually synchronized throughout the systems, they might have wildly different rates and disagree on the proper time of simultaneous events but would still agree that they occurred at the same time at both locations in their own frames. i.e:were simultaneous.
The relevant factor is synchronization. Not that the time readings in one frame don't agree with the readings of another frame but that the clocks in the other frame don't appear to agree with the other clocks in that same frame [appear asynchronous]. Of course, like most SR phenomena, this is reciprocal.
Observers in each frame assume their clocks are synchronized and the clocks in the other frame are off.

As to why , one answer seems to be in the Einstein method of synchronization.
This system, based on the speed of light and the calculated length of the light path between clocks automatically produces asynchronous clock systems if the system is in motion. In this case the receiving clock will have moved while the light is in transit so the path from the front to the back of the systems will be shorter than the path from the back to the front
So for example; if setting a clock at the back in a system that is 10 light seconds long, you would receive a time signal from the front and then assuming a travel time of 10 seconds set the clock 10 seconds ahead of the received time. But if the system has moved 2 ls. during transit the actual path is shorter and therefore the elapsed time shorter so the clock would then be running 2 seconds ahead of the sending clock at the front.

So in all cases the clocks would be running progressively later from the front to the back.
This of course is exactly what the Lorentz math regarding clock synch calculates.
The clocks in other frames are perceived to run later towards the back of the train [other frame] This can only be quantified relatively and is completely impossible to determine internally yet I believe there is compelling reason to consider this actual and relevant, and that is the invariance of measured light speed.

For example a system at rest with a clock system set up to time light speed in all directions is accelerated to a new velocity will still measure the same speed as previously.
Now of course it is not possible to say anything about the change ,not only is the difference not quantifiable but we can't even say if it is faster or slower but only that it is different. And no matter how much we accelerate to new velocities the measurement of c will remain the same. SO this is a measurement relative, not to other frames and their variable clocks and rulers, but relative to light which we consider to be actually constant and universally invariant [outside of G fields].
SO it seems to me that unless you consider these changes of velocity somehow illusory and not effecting any change of velocity relative to the light measured it is relevant to seek a rational mechanism to explain why we can't measure this change.

I have heard various attempts to do this through time dilation and length contraction, but found that not only do these not logically explain the facts, but in fact may be completely irrelevant to the question.

On the other hand asynchronous clocks with the right degree of offset does logically explain it. And since the clock convention is based on the assumption of the measured invariance it is not surprising that its application would produce the exactly right degree of asynchronicity.
Not that I am suggesting there is anything arbitrary or artificial about the the convention itself. Not only is it totally rational but as far as I can see may be the only rational system possible.
If anyone has an alternative explanation for the measured invariance I would like to hear it.

I hope this may be relevant to your question of why velocity would effect different clock times
 
Last edited:
  • #219
Austin0 said:
You seem to be missing the point:As Einstein so clearly pointed out ,unless two events occur in close proximity they cannot be determined or considered to be absolutely or actually simultaneous by any observers in any frame ,of events spatially separated.
Whether or not the universe occurs everywhere as a single simultaneous event and any pair of events is either actually simultaneous [in the sense that we normally think] or not, is moot. There is simply no way to tell and LET as well as SR are both aware of and operate on this basis. To my recollection the only reference to actual simultaneity in LET is the concept that only in a frame actually at rest could clocks be actually synchronized/simultaneous. While this idea is certainly logical and could quite possibly be correct it is absolutely useless as there is, once again, no possible means of determining the actual state of motion of any system.
SO there is no possible alternative to using the purely operative definition : occurring at the same coordinate time as indicated by local clocks. With no implication or assumption of actually or absoluteness in any case whatsoever.
It becomes a question of clock synchronization.
How do you actually synchronize clocks that are at disparate locations? Obviously somewhat a catch 22 as we know it is impossible to determine actual simultaneity for any separated events including setting clocks to the same time. Well Einstein did come up with a rational method that, while it didn't produce actual synchronization, did make physics work just fine in all frames.
Thanks for reiterating these facts in other words; that sometimes helps. :smile:
Clock rates have nothing to do with it. [..]
Not quite so: clock rates do not directly have to do with Einstein synchronization. However, in the case of clock transport, clock rates have indirectly to do with it.
As to why , one answer seems to be in the Einstein method of synchronization.
This system, based on the speed of light and the calculated length of the light path between clocks automatically produces asynchronous clock systems if the system is in motion. In this case the receiving clock will have moved while the light is in transit so the path from the front to the back of the systems will be shorter than the path from the back to the front.
[..] For example a system at rest with a clock system set up to time light speed in all directions is accelerated to a new velocity will still measure the same speed as previously. [..]
The first two sentences are correct, but your last sentence seems to contradict it... If you accelerate a system, afterwards the clocks are not synchronous anymore according to the synchronization definition; the clocks need to be re-synchronized. Perhaps you just forgot to mention that essential fact?
SO it seems to me that unless you consider these changes of velocity somehow illusory and not effecting any change of velocity relative to the light measured it is relevant to seek a rational mechanism to explain why we can't measure this change.
Again, that is wrong, so I now think that you did not just forget to mention it: the change of velocity can directly be measured inside the train on the not-yet re-synchronized clocks - you could use the system as an optical accelerometer.
I have heard various attempts to do this through time dilation and length contraction, but found that not only do these not logically explain the facts, but in fact may be completely irrelevant to the question. [..]
That remark is puzzling... what you state sounds like an SR exercise (indeed it can become rather complex), and not as a real problem.
Not that I am suggesting there is anything arbitrary or artificial about the the convention itself. Not only is it totally rational but as far as I can see may be the only rational system possible. If anyone has an alternative explanation for the measured invariance I would like to hear it. [..]
I missed your explanation (sorry). The explanations that I know are:

1. stationary ether + conservation laws
2. block universe + relativity principle
3. any combination of the above

Anyway, a discussion of "explanations" is quite a different topic as the one of this thread.
 
  • #220
DaleSpam said:
There is only one interpretation associated with the term RoS. You have failed to document that this confusion is anything other than a personal confusion unique to you and due entirely to your desire to invent some non-existent problem with SR by pretending that it is circular when it is not.

Again, I challenge you to provide any mainstream scientific reference supporting your use of the term "relativity of the time coordinate". In the absence of that, at least provide a pair of mainstream scientific references documenting that there are indeed two different uses of the term "relativity of simultaneity" which disagree with each other in the sense you are suggesting.

If you cannot find either of those then you must admit that the different interpretations associated with the term RoS are only in your mind and that such different interpretations are not part of the mainstream scientific literature.
It should be clear enough that the misunderstanding is on my part; because my application of reasoning has given rise to an understanding that is at odds with Einsteinian relativity, and insofar as ER is correct, the misunderstanding must be on my part.

My hope is, that by presenting my understanding, and the reasoning which underlies it, as clearly and cogently as possible, it will be possible to identify where the error lies and hopefully correct it; because I find it pretty frustrating trying to get my head around a lot of the stuff in the theory.

Interpretations
My understanding of RoS is that events which are simultaneous in one reference frame are not necessarily simultaneous in other reference frames i.e. if two events are not simultaneous in one reference frame it is possible that they are simultaneous in other reference frames. Is that accurate to any degree?

My understanding of Absolute Simultaneity (AS) is that if events are simultaneous in one reference frame, then they are simultaneous across all reference frames. Is this accurate at all?

From this I would deduce that RoS is not compatible with AS, because AS doesn't allow for the conditions that constitute RoS i.e. it doesn't allow for the posibility that events are simultaneous in one reference frame, but not in others. Is that a fair deduction?

My understanding of Lorentzian Relativity (LR) is that it incorporates AS. Is that accurate?

LR uses the Lorentz transform - that much seems pretty certain.

From the above I would deduce that, if LR incorporates AS and also uses the Lorentz transform; and if AS and RoS are incompatible, then RoS cannot be a consequence of the Lorentz transform; or, alternatively, there must be more than one interpretation of the term RoS. Is there an error in reasoning there?



Incidentally, if I've understood correctly, Harry has alluded to the fact that there are a number of interpretations of RoS, a few times in this thread. Is he mistaken in that, or have I taken him up incorreclty?


DaleSpam said:
It is required when you are first claiming that such a discrepancy exists in the accepted use of the term "relativity of simultaneity" and second that this discrepancy leads to special relativity being circular. You cannot weasel out of it this way and pretend on one hand that it is only a literary technique while on the other hand claiming that this literary technique demonstrates an inconsistency or weakness in a scientific theory.

If it demonstrates a problem with a scientific theory then it must be a scientific term. If it is a literary technique then it cannot demonstrate a problem with a scientific theory. You are contradicting yourself in this attempt to avoid the issue, I can only assume it is because you know full well that there is no such reference and you are aware that you are inventing the supposed conflict for your own rhetorical purposes.

I will assert once again that [itex](\text{PoR} \cap \text{C}) \rightarrow (\text{TD} \cap \text{LC} \cap \text{RoS})[/itex] as all of those terms are commonly understood in mainstream science with no circularity. (All referring to measurments only, I still am not interested in discussing non-scientific concepts like unmeasurable things here)
As mentioned above, my application of logic has lead me to an understanding which is in conflict with Einsteinian relativity; if ER is correct then the misunderstanding must be on my part. By presenting the logic that has lead to the erroneous conclusion, it is hoped that the error in understanding or the error in reasoning can be clarified.

In order to present my reasoning, as clearly and as cogently as possible, it is necessary to make use of certain literary techniques for the purpose of clarity, because there appeared to have been some confusion as to the point that was being made. The term was used for the purpose of clarifying the logic and avoiding equivocation. Citations aren't necessarily required on that basis, I presume.
 
  • #221
GeorgeDishman said:
I've reordered your response to try to address it more logically.

No, the main problem is that you are trying to use a new phrase for an old concept and then use the old phrase to mean something new. Swapping jargon terms without telling anyone what you mean will always cause confusion.
Apologies, that wasn't the intention. The intention was to try and clarify my understanding, as I was confused about the terms, and so felt the need to distinguish them to try and provide the opportunity to highlight where the issue was.



GeorgeDishman said:
I have highlighted two phrases which are the key to the confusion, there is and has only ever been one concept of simultaneity but you are trying to treat it as two. You have generated the problem in two parts, first by coining a new phrase, "relativity of time coordinates" and using that to mean what everyone else means by "relativity of simultaneity" and secondly by claiming that you have some new concept for which you re-use the existing term but without ever saying what you think it means.
I haven't be treating the concept of simultaneity as two separate concepts, at least not intentionally.

I have been trying to distinguish between two different concepts; Absolute Simultaneity (AS) and RoS. I think I've outlined my understanding of both a number of times in the thread though.

But just for clarity sake, my understanding of RoS is that events which are simultaneous in one reference frame are not necessarily simultaneous in another, such that events can be simultaneous in one reference frame and not simultaneous in another.

My understandin of AS is that events that are simultaneous in one reference frame are simultaneous in all reference frames; this would appear to be incompatible with RoS because it does not allow for the possibility that events that are not simultaneous in one reference frame, are simultaneous in another.

Those are the two concepts of simultaneity I have been using.


GeorgeDishman said:
Two events are classified as happening "simultaneously" if the times at which they occur are equal. To represent equality we use the symbol "=" and we say Ta=Tb if the numerical values of Ta and Tb are the same. Ta and Tb are of course the time coordinates of events A and B in some frame determined locally by using synchronised clocks. That method is the same for Newton's mechanics, LET and SR.
This is an area that I'm not clear on, but will address it below where you raise it again.



GeorgeDishman said:
You have been asked several times to explain what alternative meaning you would give to "simultaneity" but have never offered any definition.
Hopefully it's clear that that there is no disagreement over the term, or concept of simultaneity; the contrast was between AS and RoS, which, to my mind, appear to be incompatible.


GeorgeDishman said:
You have that the wrong way round. The term "relativity of simultaneity" is already taken and well defined, it means the equality of the times of the events allocated from syncronised local clocks. If you want to invent a new concept, what you need to do is first come up the scientific definition of that concept (i.e. how it can be measured) and secondly, to avoid the confusion you are currently creating, you need to find a new name for the new concept.
I'm not too concerned with inventing a new concept; I will settle for understanding the existing ones :smile:

Does "the equality of the times of the events allocated from syncronised local clocks" refers more to simultaneity than RoS; would RoS not mean the inequality " of the times of the events allocated from syncronised local clocks" - or at least require it in some capacity to deduce RoS; otherwise AS would be deduced, no?

Again, however, this is an area I'm not clear on. My understanding was that Lorentzian relativity incorporated absolute simultaneity, but that "local clocks" would give the same time co-ordinates as those under the ER interpretation.


GeorgeDishman said:
Yes, in fact that is true in all the theories.
Are the terms "local clock" and "local time" used in all theories?



GeorgeDishman said:
No, it is wrong (but read on before replying).

"Absolute" in this sense means "agreed by all" so "absolute simultaneity" means that, if one observer using local clocks which he considers are synchronised assigns the same time coordinate to two events, then the physics will ensure that any other observer moving relative to the first will also assign equal coordinates using clocks local to the events which he considers synchronised. That would happen if the transforms of Gallilean Relativity applied, it doesn't happen under the Lorentz Transforms. Newton's world model did exhibit absolute simultaneity, those of Lorentz and Einstein do not.
Are you certain about that?

The impression that I got from discussing it with people on here was that "absolute", in LR, didn't refer to "agreed by all", rather, observers could disagree about the simultaneity of events, on the basis of the time co-ordinates, but this was due to the mechanics of the clocks meaning the clocks ran at different rates.

The effect, of the different times displayed by clocks, would be the same in both LR and ER, but the explanation for why this happens would be different. ER would conclude that time is relative, and the different times displayed by the clocks i.e. the different time co-ordinates, suggest the RoS of events.

On the other hand, LR would suggest that it is the mechanics of the clock which cause it to run slow, as opposed to time being relative, and so, despite the different time co-ordinates provided by the clocks, events would still be absolutely simultaneous, because of the underlying absolute time, which "local clocks" do not display.

We might have to defer to someone else on that one.
 
  • #222
GeorgeDishman said:
Many people do but this forum exists precisely for that reason. I've tried to address the other points in your post quickly so we can get them out of the way, this aspect is where we need to put the real effort if you want to break the logjam and start understanding SR. I've got to do other stuff now but I'll try to get back to this later or maybe tomorrow. I want to do some work that will help as it comes up often but it will take some time.

Cheers George - much appreciated!
 
  • #223
harrylin said:
There are different views about the hidden reality that may explain such things as relativity of simultaneity, time dilation etc. However, the definitions of SR are purely operational definitions, as indicated in Einstein's 1905 paper.
OK, but we're not solely considering the definition of SR, we're considering them in contrast to Lorentzian relativity.

harrylin said:
I'm afraid that here it shows that you have not performed enough Lorentz transformations - if at all! The Lorentz transformations describe how physical events are mapped from one co-ordinate system to the other. Thus, there is no difference between these things; moreover, introducing a different term with the same meaning cannot help for the sake of distinguishing interpretations.
To my understanding, the difference lies in the concepts of absolute simutaneity and RoS, but the point is raised below, so I'll repsond to it there.


harrylin said:
That may be the cause: try again with substituting velocity (you said that you understood this, but evidently you did not!). You will then realize that you are prey to the exact same confusion as is possible between "absolute velocity" and "velocity is absolute". Similarly, "absolute simultaneity" has nothing to do with "simultaneity is absolute", contrary to what you say here above.
My understanding is that simultaneity refers to the case where events happen at the same time in one reference frame.

RoS refers to where events that are simultaneous in one reference frame are not necessarily simultaneous in all referene frame i.e they can be simultaneous in one, but not in another.

Absolute simultaneity refers to where events are simultaneous in all reference frames.


I can't make the connection with the velocity comparison, because I'm not familiar with any other conceptualisation of absolute velocity. Would you be able to outline a conceptualisation of AS, that is different from the one I've stated?


harrylin said:
That misleading picture has been corrected by all the discussions that you had with people in this thread - right?
Not as of yet; the concept of RoS that I've been working with has been reiterated numerous times by people in this thread, but you're the first to suggest that the conceptualisation of absolute simultaneity I've been working off is incorrect.

As mentioned, I'm not familiar with a conceptualisation of AS other than the one I've outlined above.


harrylin said:
I repeat, for quadruple clarity: the RoS of physical events is compatible with absolute simultaneity, just as the relativity of physical velocities is compatible absolute velocity.
According to the descriptions of both RoS and AS that I am familiar with, this statement is inaccurate, as I believe I have demonstrated logically.

If there is an alternative conceptualisation of AS, then the answer might lie in that; as of yet no one has outlined such a conceptualisation, that would be compatible with RoS.


harrylin said:
Sure - although historically RoS was first.
How is this possible? How did RoS come before the Lorentz transform if RoS is a consequence of it?

harrylin said:
RoS is not an interpretation, it follows directly from a well defined operational definition of simultaneity, based on position coordinates and clocks! You can read it here:
http://www.fourmilab.ch/etexts/einstein/specrel/www/
Does it refer to the case where events that are simultaneous in one reference frame, are not necessarily simultaneous in another i.e. they can be simultaneous in one, and not simultaneous in another?

Is this different from the concept of Absolute Simultaneity?

Does Lorentzian relativity incorporate AS?

If my understanding of AS is incorrect, what is the conceptualisations of AS that is compatible with RoS; or how is it similar to the velocity example you gave?



harrylin said:
He referred to it as "local time" already before Einstein came around and I don't think that he adapted his jargon to that of Einstein. Moreover, he called "absolute time" "true time" or "universal time". You can read it here:
http://en.wikisource.org/wiki/The_Einstein_Theory_of_Relativity
What is the difference between Lorentz's "local time" and the time measured by clocks in ER?

If events are simultaneous in "true time", according to LR, are they simultaneous for all observers, even if those observers don't think they are?



harrylin said:
I do worry, and with ample reason: you keep confounding clock rate with relativity of simultaneity... so again, no: The indication of different times in Einstein's train illustration is not due to different clock rates.
The point isn't that RoS is due to different clock rates.

If one clock ticks slower than another clock because of it's relative motion, then that clock is said to tick at a slower rate. If clocks tick at different rates then they will indicate different times.

That is my understanding at least; is there an issue with the logic?


harrylin said:
I now gave you a link to a presentation by Lorentz here above.
and from a quick search of recent discussions:
https://www.physicsforums.com/showthread.php?p=3558212
I read the presentation by Lorentz, but I didn't see much in the way of discussion of Ros with repsect to Lorentzian relativity.

I will check out that thread tomorrow, hopefully, bcos it is late here, and I'm going to head to bed now.


Thanks for your considered reply though, it suggests that there might be a point of clarification which will help me better understand this; namely, the conceptualisation of AS that is different to the one I'm familiar with and is compatible with RoS.
 
  • #224
mangaroosh said:
Interpretations
My understanding of RoS is that events which are simultaneous in one reference frame are not necessarily simultaneous in other reference frames i.e. if two events are not simultaneous in one reference frame it is possible that they are simultaneous in other reference frames. Is that accurate to any degree?
Yes, it's accurate, but it's like saying that since siblings can't get married, if two people aren't siblings then it is possible they are married.
mangaroosh said:
My understanding of Absolute Simultaneity (AS) is that if events are simultaneous in one reference frame, then they are simultaneous across all reference frames. Is this accurate at all?
No. That's like saying that all the people who aren't siblings are married to each other.
mangaroosh said:
From this I would deduce that RoS is not compatible with AS, because AS doesn't allow for the conditions that constitute RoS i.e. it doesn't allow for the posibility that events are simultaneous in one reference frame, but not in others. Is that a fair deduction?
No. That's like saying that since siblings can't get married, people who have siblings can't get married.
mangaroosh said:
My understanding of Lorentzian Relativity (LR) is that it incorporates AS. Is that accurate?
Only in one preferred frame. LR is simply the notion that one reference frame is preferred. The synchronized coordinate clocks stationary in that one reference frame provide absolute time. All other clocks moving or that have not been synchronized do not display absolute time.
mangaroosh said:
LR uses the Lorentz transform - that much seems pretty certain.
LR did not exist prior to 1905 and the LT that was used prior to 1905 is not the same one that we use today. I told you to look up the history of LT in wikipedia. You should realize this by now.
mangaroosh said:
From the above I would deduce that, if LR incorporates AS and also uses the Lorentz transform; and if AS and RoS are incompatible, then RoS cannot be a consequence of the Lorentz transform; or, alternatively, there must be more than one interpretation of the term RoS. Is there an error in reasoning there?
Yes. Prior to Einstein, nobody thought in terms of time being relative. Everybody thought that there only existed an absolute time throughout the entire universe. If a clock didn't tick at the same rate as the absolute time because it was moving, that was blamed on the mechanics of the clock, nobody ever dreamed that time itself could be anything other than absolute. With absolute time, it was possible to assert that the times of events would only be validly determined if the absolute time was used to describe those events.

Scientists were well aware of the fact that actual real ticking clocks would tick at different rates when they were moving with respect to one another and therefore concluded that since we on the Earth were constantly changing our state of motion, the chances are our clocks never ticked in sync with absolute time. They thus interpreted real ticking clocks to be displaying what they called "local time" instead of using the term we now use which is "relative time" from which we derive "relativity of simultaneity". Prior to Einstein nobody ever would have thought to use the term RoS because they weren't thinking in terms of time being relative.

So prior to Einstein, there was no such thing as LR and no such thing as RoS and the LT was different, in fact it kept changing. LET asserted AS. But after Einstein, LR got invented for the first time by using the same LT that SR used and it is usually what we call LET nowadays but it's important to realize that this concept of LET did not exist prior to Einstein.

So now if you want to promote LR, you are simply saying that it is mathematically identical to SR but instead of adopting Einstein's second postulate, it adopts the postulate that light propagates at c only in the preferred frame. It would assert that AS exists only in that one preferred frame and that RoS exists in all other frames moving with respect to that one preferred frame.
 
  • #225
mangaroosh said:
It should be clear enough that the misunderstanding is on my part; because my application of reasoning has given rise to an understanding that is at odds with Einsteinian relativity, and insofar as ER is correct, the misunderstanding must be on my part.

My hope is, that by presenting my understanding, and the reasoning which underlies it, as clearly and cogently as possible, it will be possible to identify where the error lies and hopefully correct it; because I find it pretty frustrating trying to get my head around a lot of the stuff in the theory.
OK, that is completely reasonable. However, I would encourage you to NOT invent new terminology for existing concepts nor to invent new concepts to apply to existing terminology. Either of those actions will be counterproductive to your stated goals.

Furthermore, if you want to understand ER (or modern science in general) you need to drop your fascination with the distinction between "actual" values and measured values. Unmeasurable values cannot be investigated using the scientific method and so ER deliberately discards such. My comments below refer to measured values, unless explicitly indicated.

mangaroosh said:
My understanding of RoS is that events which are simultaneous in one reference frame are not necessarily simultaneous in other reference frames i.e. if two events are not simultaneous in one reference frame it is possible that they are simultaneous in other reference frames. Is that accurate to any degree?
Yes. Note that this definition requires multiple frames.

mangaroosh said:
My understanding of Absolute Simultaneity (AS) is that if events are simultaneous in one reference frame, then they are simultaneous across all reference frames. Is this accurate at all?
Yes. Note that this definition requires multiple frames.

mangaroosh said:
From this I would deduce that RoS is not compatible with AS, because AS doesn't allow for the conditions that constitute RoS i.e. it doesn't allow for the posibility that events are simultaneous in one reference frame, but not in others. Is that a fair deduction?
Agreed.

mangaroosh said:
My understanding of Lorentzian Relativity (LR) is that it incorporates AS. Is that accurate?
No. Recall that the above definition of AS and RoS required the comparison between multiple reference frames. In LR different reference frames (whether the unique but unmeasurable aether frame or any of the many measurable local frames) are related by the Lorentz transform which clearly introduces RoS between any two frames.

Note, understanding of LR is not needed for understanding of SR. In fact, LR is a poorly-defined theory that was dropped more than a century ago and has not been worked on much. There is AFAIK no one "definitive" source for LR, so asking about LR is likely to provoke different responses from different people, which will detract from your stated goal of understanding SR.

mangaroosh said:
LR uses the Lorentz transform - that much seems pretty certain.
Yes.

mangaroosh said:
From the above I would deduce that, if LR incorporates AS and also uses the Lorentz transform; and if AS and RoS are incompatible, then RoS cannot be a consequence of the Lorentz transform; or, alternatively, there must be more than one interpretation of the term RoS. Is there an error in reasoning there?
Yes, LR does not incorporate AS, and LR is not helpful to understanding SR.

mangaroosh said:
As mentioned above, my application of logic has lead me to an understanding which is in conflict with Einsteinian relativity; if ER is correct then the misunderstanding must be on my part. By presenting the logic that has lead to the erroneous conclusion, it is hoped that the error in understanding or the error in reasoning can be clarified.

In order to present my reasoning, as clearly and as cogently as possible, it is necessary to make use of certain literary techniques for the purpose of clarity, because there appeared to have been some confusion as to the point that was being made. The term was used for the purpose of clarifying the logic and avoiding equivocation. Citations aren't necessarily required on that basis, I presume.
No, it is not necessary to invent new terminology nor to change the definitions of existing terminology in order to understand a theory. You will be much better served to learn and use the existing terminology and definitions.

In addition, it is against the rules of the forum to do so unless you can produce a mainstream scientific reference supporting your term. Remember that this site is primarily educational in nature, and other people besides you will read what is written here. Your introduction of new terms and redefinition of existing terms will not help you learn and will cause unncessary confusion to others reading.
 
Last edited:
  • #226
mangaroosh, do you have any questions or confusions about SR itself that do not involve comparisons with or concepts from LR? If not, then your stated goals are accomplished and all you have to do is let go of LR like the rest of the scientific community did over a century ago.
 
  • #227
mangaroosh said:
To avoid derailing this thread, it might be worth posting in the "at rest" thread.

But just to address the point raised: a consequence of the PoR says that observers can't determine if they are in motion or at rest ...

No, what it says is that all non-degenerate inertial coordinate systems are equally valid. Remember relativity is geometric so think of it in those terms: on a spacetime diagram the velocity of one object is the angle between their worldlines when they cross. What the PoR is saying is that it is meaningless to ask what the "angle between" is if you only have one line. In fact the same was true in the Newtonian world because it included Galilean Relativity but you can't use the geometric view.

Yes, and you will also need to have a grasp of "general covariance" and the effect of diffeomorphisms which is why I suggest keeping it separate.

Those concepts aren't necessarily essential to understanding that the PoR can be extended to accelerating reference frames though, would they?

Right, the context has been lost. Those will be important if you try to understand the Hole Argument regarding substantivalism, they are not relevant here but will be if you discuss it in the philosophy forum.
 
  • #228
mangaroosh said:
I am certainly interested in learning how it all works, but on the way to developing that understanding there are certain things which provide stumbling blocks, usually linked to a more intuitive understanding. Of course, an intuitive understanding isn't necessarily correct, but it usually takes reasoning to overcome it.

True, often the intuitive view is misleading but in the case of SR it actually works quite well. What is necessary though is to replace the Newtonian view of time with something rather different. Once you get the hang of that, it all falls into place and is surprisingly elegant. I want to put together something that I can use in the future as this keeps coming up but it may take some time. If I can't do it soon, I'll try to give a verbal explanation here but give me a day or two yet please.

The idea that physical effects are not explained in terms of a physical cause is particularly jarring; for example what is the explanation for the increase of mass with speed, under Einsteinian relativity; and what is the explanation for the physical contractions?

In relativity, mass is invariant. It is one of those odd quirks of history that the effect known as "relativistic mass" as a physical change only happens in aether theory!

Also, the, perhaps, most mainstream alternative explanation i.e. the spacetime explanation (or block universe), seems to require other assumptions also, with regard to time in general, specifically past and future, which don't seem to correspond to empirical experience.

Yes, that's a different ball game entirely. The block universe is a philosophical interpretation which only takes an input from relativity. There's a thread running on that in the philosophy group regarding A-series or B-series.
 
  • #229
mangaroosh said:
I haven't be treating the concept of simultaneity as two separate concepts, at least not intentionally.

OK, that has been the impression I was getting, my apologies if I got that wrong.

I have been trying to distinguish between two different concepts; Absolute Simultaneity (AS) and RoS. I think I've outlined my understanding of both a number of times in the thread though.

But just for clarity sake, my understanding of RoS is that events which are simultaneous in one reference frame are not necessarily simultaneous in another, such that events can be simultaneous in one reference frame and not simultaneous in another.

Correct.

My understandin of AS is that events that are simultaneous in one reference frame are simultaneous in all reference frames; this would appear to be incompatible with RoS because it does not allow for the possibility that events that are not simultaneous in one reference frame, are simultaneous in another.

Correct.

Those are the two concepts of simultaneity I have been using.

In that case there is no problem the key is that while "relative" and "absolute" are different, both phrases refer to the same concept of simultaneity. That's what you said above so were are good up to here.

Two events are classified as happening "simultaneously" if the times at which they occur are equal. To represent equality we use the symbol "=" and we say Ta=Tb if the numerical values of Ta and Tb are the same. Ta and Tb are of course the time coordinates of events A and B in some frame determined locally by using synchronised clocks. That method is the same for Newton's mechanics, LET and SR.

This is an area that I'm not clear on, but will address it below where you raise it again.

...

Hopefully it's clear that that there is no disagreement over the term, or concept of simultaneity

Is there or isn't there? You just said you weren't clear on it and then you say that it is clear there is no disagreement? This is very important because that is the single definition of simultaneity which the previous phrases share (and which we address later).

I'm not too concerned with inventing a new concept; I will settle for understanding the existing ones :smile:

Well you've got three of us all telling what the definitions are so that shouldn't be a problem.

Does "the equality of the times of the events allocated from syncronised local clocks" refers more to simultaneity than RoS ...

It is the definition of simultaneity which is common to both the relative and absolute alternatives. I'll say it again slowly: the word "simultaneous" means "at the same time". "The same" in mathematically means "having equal values". The values assigned against a axis are called "coordinates" so "simultaneous" means "having the same time coordinate values".

Whether the times so allocated from local clocks are "equal for all observers if equal for one" or "equal for one but not for others in motion relative to the first" is the difference between absolute and relative simultaneity.

Again, however, this is an area I'm not clear on. My understanding was that Lorentzian relativity incorporated absolute simultaneity, but that "local clocks" would give the same time co-ordinates as those under the ER interpretation.

Local clocks are used in all the theories because otherwise you introduce an unknown component concerning the signal travel time between the clock and that which is being timed. Lorentzian philosophy included presentism but in the scientific theory clocks behaved identically to those in relativity.

Are the terms "local clock" and "local time" used in all theories?

No, generally we just talk of clocks and time, it is the method that matters and that's always the same. A time is what you read from a clock.

"Absolute" in this sense means "agreed by all" so "absolute simultaneity" means that, if one observer using local clocks which he considers are synchronised assigns the same time coordinate to two events, then the physics will ensure that any other observer moving relative to the first will also assign equal coordinates using clocks local to the events which he considers synchronised. That would happen if the transforms of Gallilean Relativity applied, it doesn't happen under the Lorentz Transforms. Newton's world model did exhibit absolute simultaneity, those of Lorentz and Einstein do not.

Are you certain about that?

Yes, and so are you :wink:. Look back at what you said at the top of your reply:

my understanding of RoS is that events which are simultaneous in one reference frame are not necessarily simultaneous in another, such that events can be simultaneous in one reference frame and not simultaneous in another.

My understandin of AS is that events that are simultaneous in one reference frame are simultaneous in all reference frames?

The two statements are equivalent, agreed?

We might have to defer to someone else on that one.

I don't think there's any need now. Having resolved that, I would suggest you lay LET aside for the moment, it's philosophy is diametrically opposed to what you need to learn if you are to understand SR and will make it very difficult for you. You can always come back to it once you grasp SR, that's what I did.
 
  • #230
GeorgeDishman said:
Having resolved that, I would suggest you lay LET aside for the moment, it's philosophy is diametrically opposed to what you need to learn if you are to understand SR and will make it very difficult for you. You can always come back to it once you grasp SR, that's what I did.
I agree with this recommendation. LET is a defunct theory that was never fully fleshed out. Studying it will not help learn SR.
 
Last edited:
  • #231
mangaroosh said:
OK, but we're not solely considering the definition of SR, we're considering them in contrast to Lorentzian relativity.
As Lorentz's concise statement showed, it is mistake to think of "in contrast".
Instead, simply put: SR + ether = LR.
Similarly: SR + block universe = MR.
[..] Absolute simultaneity refers to where events are simultaneous in all reference frames.
[..] I can't make the connection with the velocity comparison, because I'm not familiar with any other conceptualisation of absolute velocity. Would you be able to outline a conceptualisation of AS, that is different from the one I've stated?
"Absolute simultaneity" commonly refers to the same unique reference frame as "absolute velocity". See aslo:
http://en.wikipedia.org/wiki/Absolute_time_and_space
I hope that you see that that resolves all confusion on this matter. :smile:
[..]
How is this possible? How did RoS come before the Lorentz transform if RoS is a consequence of it?
I'm quite sure that I let you read the explanation from the original paper in a discussion here not long ago... Now I have no time to search it back again but in a nutshell, it was already applied before time dilation was added to the transformations of Lorentz.
Does it refer to the case where events that are simultaneous in one reference frame, are not necessarily simultaneous in another i.e. they can be simultaneous in one, and not simultaneous in another?
Yes - exactly! Didn't you read it? :uhh: If you had searched for it with "simultaneous" you would have found it in a minute... and the first pages of that article are much shorter than the total of pages that you had us write here in discussions with you...
As a matter of fact, if you don't paste that passage in your next reply to me, I will not reply to you for at least one month. :grumpy:
[..] Does Lorentzian relativity incorporate AS?
Yes, but perhaps not explicitly: he referred to it as "true time".
[..] What is the difference between Lorentz's "local time" and the time measured by clocks in ER?
No difference after 1905 (before 1905 Lorentz did not have a clear idea about it, but Poincare already applied it to the time measured by clocks).
If events are simultaneous in "true time", according to LR, are they simultaneous for all observers, even if those observers don't think they are?
Definitely not: observers can only use "local time", since they cannot know true time. :tongue2:
[..] If one clock ticks slower than another clock because of it's relative motion, then that clock is said to tick at a slower rate. If clocks tick at different rates then they will indicate different times.

That is my understanding at least; is there an issue with the logic?
1. That has nothing to do with Einstein's train illustration which you claimed to understand and which you claimed to comment on - thus your claim was totally wrong. So, please explain Einstein's train example, as a test. :devil:
2. Your logic appears very incomplete to me - but that's irrelevant at this point.
I read the presentation by Lorentz, but I didn't see much in the way of discussion of Ros with repsect to Lorentzian relativity.
Here's again the key section that you can't have missed (but of which you did read the context so that you surely won't misunderstand it) - and funny enough, it appears that the first section wasn't authored by Lorentz:

(1)"it was necessary incidentally to throw over the one universal time, and substitute local times attached to moving bodies and varying according to their motion. The equations on which the theory of relativity is based are due to Lorentz, but Einstein connected them with his general principle, namely, that there must be nothing, in observable phenomena, which could be attributed to absolute motion of the observer. [..] In orthodox Newtonian dynamics the principle of relativity had a simpler form, which did not require the substitution of local time for general time. " [..]
(2)"It is not necessary to give up entirely even the ether. [..] In my opinion it is not impossible that in the future this road, indeed abandoned at present, will once more be followed with good results, if only because it can lead to the thinking out of new experimental tests. Einstein's theory need not keep us from so doing; only the ideas about the ether must accord with it."

Now, the "time" of a system in rest with the ether is often called "absolute" or "true" time, consistent with Newton's defnitions. I hope that it is now clear to you that such a "true time" is not at all incompatible with RoS, just as also our "universal time" is not incompatible with RoS - even if astronauts use it. As a matter of fact, astronauts could use both local and universal time, and thus use "dual time" - that would be a neat example of RoS. :rolleyes:

I will check out that thread tomorrow, hopefully, bcos it is late here, and I'm going to head to bed now.
Good! :smile:
Thanks for your considered reply though, it suggests that there might be a point of clarification which will help me better understand this; namely, the conceptualisation of AS that is different to the one I'm familiar with and is compatible with RoS.
Yes indeed! And as I mentioned before, a lot of expressions had acquired multiple compatible meanings before SR, which became incompatible afterwards. Consequently, one often has to consider the context to understand what a writer means when using certain words.
 
  • #232
harrylin said:
i hope so, and perhaps the clearest answer is to simply correct a wrong sentence.
For example, not:

"RoTC can be interpreted in two (or maybe more) ways; it can be interpreted in such a way that incorporates absolute simultaneity, or it can be interpreted in such a way that RoS is incorporated."

But:

"The existence of RoS can be interpreted in two (or maybe more) ways; it can be interpreted independently of absolute simultaneity, or it can be interpreted in such a way that only RoS exists."

And now they are understood to be compatible. :smile:
Apologies, I don't fully grasp this point; possibly because of the concept of absolute simultaneity.

As mentioned, my understanding of AS is that events simultaneous in one reference frame are simultaneous across all reference frames.

This is contrasted with my understanding of RoS, where events that are simultaneous in one reference frame are not simultaneous in another.

The two concepts don't appear to be compatible, because AS doesn't allow for simultaneous events [in one reference frame] not to be simultaneous in another.

For this reason, there must be another interpretation of AS, that I'm not aware of, that is compatible with RoS.



harrylin said:
Compare:

"Relative velocity can be interpreted in two (or maybe more) ways; it can be interpreted independently of (or even due to) absolute velocity, or it can be interpreted in such a way that only relative velocities exist."

PS. what may be confusing is that words that had multiple but compatible meanings before relativity, acquired incompatible meanings from then on (e.g. "absolute"). As a result, sometimes people use the same words or phrases to mean something different, and at other times people say something very different to mean the same.
I don't see the similarities in this case, but if you could elaborate and draw a more direct comparison, then I might be able to see it. My reasoning would lead me to question how only relative velocities can exist, without absolute motion; not necessarily absolute velocity, which might be a contradiction in terms. This is probably a point better suited to the "at rest" thread though.

You mention about words that had compatible meanings before relativity but not after, and indicate the term "absolute"; how did "absolute" acquire incompatible meanings after relativity, and what does it mean now?

Also, is there an issue in my understanding of "absolute simultaneity" as outlined above?
 
  • #233
Austin0 said:
You seem to be missing the point:As Einstein so clearly pointed out ,unless two events occur in close proximity they cannot be determined or considered to be absolutely or actually simultaneous by any observers in any frame ,of events spatially separated.
Whether or not the universe occurs everywhere as a single simultaneous event and any pair of events is either actually simultaneous [in the sense that we normally think] or not, is moot. There is simply no way to tell and LET as well as SR are both aware of and operate on this basis. To my recollection the only reference to actual simultaneity in LET is the concept that only in a frame actually at rest could clocks be actually synchronized/simultaneous. While this idea is certainly logical and could quite possibly be correct it is absolutely useless as there is, once again, no possible means of determining the actual state of motion of any system.
SO there is no possible alternative to using the purely operative definition : occurring at the same coordinate time as indicated by local clocks. With no implication or assumption of actually or absoluteness in any case whatsoever.
It becomes a question of clock synchronization.
How do you actually synchronize clocks that are at disparate locations? Obviously somewhat a catch 22 as we know it is impossible to determine actual simultaneity for any separated events including setting clocks to the same time. Well Einstein did come up with a rational method that, while it didn't produce actual synchronization, did make physics work just fine in all frames.
Hey Austin, thanks for the reply.

The understanding of LET that I have picked up in other threads, is that the hypothetical rest frame is the "true time" frame which determines whether or not events are actually simultaneous. "Local clocks" do not tell the "true time" so the time co-ordinates they provide cannot be used for determining the simultaneity of events. Observers might disagree about whether events are simultaneous or not, but they essentially cannot tell; however, the concept of absolute simultaneity appears to be incorporated on the basis of this absolute frame.

The point about clock synchronisation, and clocks in general, is unfortunately one I don't think I can go into, because it might be deemed to be too close to the topic of a thread that was locked in the General discussion thread - How does a clock measure time?. I would need confirmation from a mentor that it is alright to discuss the issue.

I think it should be alright to pose the question in terms of LET though, and ask, how can we determine that Lorentz's "local clock" hypothesis is not the correct one? If it is correct, then it means that RoS cannot be determined either, because "local clocks" do not tell the "true time" and so cannot be used to determine the simultaneity of events.



Austin0 said:
Clock rates have nothing to do with it. If two systems have clocks that are actually synchronized throughout the systems, they might have wildly different rates and disagree on the proper time of simultaneous events but would still agree that they occurred at the same time at both locations in their own frames. i.e:were simultaneous.
Apologies, I don't fully get this point.

Does this mean that there are synchronised clocks in two reference frames, with wildly different rates?

Austin0 said:
The relevant factor is synchronization. Not that the time readings in one frame don't agree with the readings of another frame but that the clocks in the other frame don't appear to agree with the other clocks in that same frame [appear asynchronous]. Of course, like most SR phenomena, this is reciprocal.
Observers in each frame assume their clocks are synchronized and the clocks in the other frame are off.

As to why , one answer seems to be in the Einstein method of synchronization.
This system, based on the speed of light and the calculated length of the light path between clocks automatically produces asynchronous clock systems if the system is in motion. In this case the receiving clock will have moved while the light is in transit so the path from the front to the back of the systems will be shorter than the path from the back to the front
So for example; if setting a clock at the back in a system that is 10 light seconds long, you would receive a time signal from the front and then assuming a travel time of 10 seconds set the clock 10 seconds ahead of the received time. But if the system has moved 2 ls. during transit the actual path is shorter and therefore the elapsed time shorter so the clock would then be running 2 seconds ahead of the sending clock at the front.

So in all cases the clocks would be running progressively later from the front to the back.
This of course is exactly what the Lorentz math regarding clock synch calculates.
The clocks in other frames are perceived to run later towards the back of the train [other frame] This can only be quantified relatively and is completely impossible to determine internally yet I believe there is compelling reason to consider this actual and relevant, and that is the invariance of measured light speed.

For example a system at rest with a clock system set up to time light speed in all directions is accelerated to a new velocity will still measure the same speed as previously.
Now of course it is not possible to say anything about the change ,not only is the difference not quantifiable but we can't even say if it is faster or slower but only that it is different. And no matter how much we accelerate to new velocities the measurement of c will remain the same. SO this is a measurement relative, not to other frames and their variable clocks and rulers, but relative to light which we consider to be actually constant and universally invariant [outside of G fields].
SO it seems to me that unless you consider these changes of velocity somehow illusory and not effecting any change of velocity relative to the light measured it is relevant to seek a rational mechanism to explain why we can't measure this change.

I have heard various attempts to do this through time dilation and length contraction, but found that not only do these not logically explain the facts, but in fact may be completely irrelevant to the question.

On the other hand asynchronous clocks with the right degree of offset does logically explain it. And since the clock convention is based on the assumption of the measured invariance it is not surprising that its application would produce the exactly right degree of asynchronicity.
Not that I am suggesting there is anything arbitrary or artificial about the the convention itself. Not only is it totally rational but as far as I can see may be the only rational system possible.
If anyone has an alternative explanation for the measured invariance I would like to hear it.

I hope this may be relevant to your question of why velocity would effect different clock times
This is indeed relevant Austin, cheers. As mentioned, it is also relevant to the locked thread linked to above, which I would need confirmation of that it is alright to go into.
 
  • #234
ghwellsjr said:
Yes, it's accurate, but it's like saying that since siblings can't get married, if two people aren't siblings then it is possible they are married.
I don't think the analogy is accurate.

Simultaneous means that events happen at the same time.

RoS means that they can happen at the same time in one reference frame, but that they happen at different times in another reference frame.

Marriage is where two people are joined in matrimony. RoM (relativity of marriage) would be where two people are joined in matrimony in one reference frame but not in another.


ghwellsjr said:
No. That's like saying that all the people who aren't siblings are married to each other.
Not necessarily, it would be like saying two people who are married in one reference frame are married in all reference frames - absolute marriage.


ghwellsjr said:
No. That's like saying that since siblings can't get married, people who have siblings can't get married.
I would be more inclined to say that absolute marriage is not compatible with RoM because it doesn't allow for the possibility of two people being married in one reference frame but unmarried in another.


ghwellsjr said:
Only in one preferred frame. LR is simply the notion that one reference frame is preferred. The synchronized coordinate clocks stationary in that one reference frame provide absolute time. All other clocks moving or that have not been synchronized do not display absolute time.
My understanding of this would be that "local clocks", therefore, cannot be used to determine the simultaneity of events. Only the absolute frame can determine if events are simultaneous or not. If events are simultaneous in the absolute rest frame, then they are simultaneous in all reference frames even if the observers disagree because of the time co-ordinates provided by their inaccurate clocks.


ghwellsjr said:
LR did not exist prior to 1905 and the LT that was used prior to 1905 is not the same one that we use today. I told you to look up the history of LT in wikipedia. You should realize this by now.
OK, but the current state of affairs is that LR uses the LT, no?


ghwellsjr said:
Yes. Prior to Einstein, nobody thought in terms of time being relative. Everybody thought that there only existed an absolute time throughout the entire universe. If a clock didn't tick at the same rate as the absolute time because it was moving, that was blamed on the mechanics of the clock, nobody ever dreamed that time itself could be anything other than absolute. With absolute time, it was possible to assert that the times of events would only be validly determined if the absolute time was used to describe those events.

Scientists were well aware of the fact that actual real ticking clocks would tick at different rates when they were moving with respect to one another and therefore concluded that since we on the Earth were constantly changing our state of motion, the chances are our clocks never ticked in sync with absolute time. They thus interpreted real ticking clocks to be displaying what they called "local time" instead of using the term we now use which is "relative time" from which we derive "relativity of simultaneity". Prior to Einstein nobody ever would have thought to use the term RoS because they weren't thinking in terms of time being relative.

So prior to Einstein, there was no such thing as LR and no such thing as RoS and the LT was different, in fact it kept changing. LET asserted AS. But after Einstein, LR got invented for the first time by using the same LT that SR used and it is usually what we call LET nowadays but it's important to realize that this concept of LET did not exist prior to Einstein.

So now if you want to promote LR, you are simply saying that it is mathematically identical to SR but instead of adopting Einstein's second postulate, it adopts the postulate that light propagates at c only in the preferred frame. It would assert that AS exists only in that one preferred frame and that RoS exists in all other frames moving with respect to that one preferred frame.
If "local clocks" cannot determine "true time", and if their different times are the result of mechanics, then they presumably cannot be used to determine the simultaneity of events, in any meaningful way.

If events are simultaneous in the absolute rest frame, then presumably they must be simultaneous in all reference frames, even if the observers cannot determine this to be so, due to their inaccurate clocks. They might disagree on whether events are simultaneous or not, but presumably they can't really say; and instead of the Einsteinian notion that both reference frames are equal, it would be possible that, according to LET, that one observer actually is correct, because they might actually be in the absolute rest frame, just unable to determine it.


It might be worth mentioning that, if time did not exist, a notion mentioned in this, mainstream source, then it would arguably be pretty much identical to the notion of absolute time, insofar as presentism would be prevalent. In that instance, the "relative time" of local clocks could not be used to determine RoS.

I don't think that is how LET is formulated presently though is it? Although I think the notion would be more compatible with LET than Einsteinian relativity.
 
  • #235
DaleSpam said:
OK, that is completely reasonable. However, I would encourage you to NOT invent new terminology for existing concepts nor to invent new concepts to apply to existing terminology. Either of those actions will be counterproductive to your stated goals.
Apologies, the intention wasn't to invent new terminology, but to try and clarify my understanding; as we seemed to be at cross purposes, I thought it would be helpful to make a distinction between, what appeared to me as, different interpretations.


DaleSpam said:
Furthermore, if you want to understand ER (or modern science in general) you need to drop your fascination with the distinction between "actual" values and measured values. Unmeasurable values cannot be investigated using the scientific method and so ER deliberately discards such. My comments below refer to measured values, unless explicitly indicated.
I tend to find that I develop a better understanding of something when I subject it to critical reasoning; the distinction between actual and measured values came about on the basis of that reasoning. It also seems to be an important distinction, again for the reasoning that was outlined. Again, based on logical reasoning, it would seem that RoS is a consequence of the actual speed of light remaining invariant, but the actual speed of light would be unmeasurable, given the distinction between actual and measured speeds.

As much as I would like to eschew reason in my attempts to understand relativity, it isn't something that comes naturally.


DaleSpam said:
Yes. Note that this definition requires multiple frames.

Yes. Note that this definition requires multiple frames.

Agreed.
OK, so it seems we are agreed on that much. The questions that remains are, is AS incorporated in LR; and is the Einsteinian interpretation of RoS the same as the Lorentzian?

Which you raise below.

DaleSpam said:
No. Recall that the above definition of AS and RoS required the comparison between multiple reference frames. In LR different reference frames (whether the unique but unmeasurable aether frame or any of the many measurable local frames) are related by the Lorentz transform which clearly introduces RoS between any two frames.
This is where the point of contention lies. According to LR, only the absolute rest frame has the "true time", so presumably only this frame can be used to determine the simultaneity of events. If events are simultaneous in the absolute rest frame, then they are presumably simultaneous across all reference frames, except that observers will disagree over this because the time co-ordinates of the events, provided by their clocks, will differ. But the time co-ordinates of the "local clocks" are not the "true time" co-ordinates of the events, so they cannot, necessarily, be used to determine the simultaneity of events.

That would be my understanding.


DaleSpam said:
Note, understanding of LR is not needed for understanding of SR. In fact, LR is a poorly-defined theory that was dropped more than a century ago and has not been worked on much. There is AFAIK no one "definitive" source for LR, so asking about LR is likely to provoke different responses from different people, which will detract from your stated goal of understanding SR.
You have mentioned yourself, in other threads, that there is no experimental difference between SR and LR, that it is essentially a matter of philosophy - or logic, perhaps - as to which one is preferred. Given the parity of the two theories in terms of experimental support, I think it would be helpful to contrast the two and thereby gain a better understanding of both. Surely where two theories are experimentally equivalent to one another theory, both theories must be someway understood before one can be rejected in favour of the other? How else could it be determined that they are experimentally equivalent?


DaleSpam said:
Yes, LR does not incorporate AS, and LR is not helpful to understanding SR.
LR can be helpful in understanding SR, insofar as it provides an alternative interpretation against which to compare SR and perhaps identify some of the underlying assumptions, expressed and tacit, which need to be understood to fully understand the theory.

As for LR incorporating AS, that is, again, a bone of contention. Given the postulation of "true time" or "universal time", and the contention that "local clocks" do not tell the "true time", presumably the time co-ordinates of "local clocks" cannot be used to determine the simultaneity of events, in any meaningful way? If the time co-ordinates differ then, according to LR, it is because the "local clocks" are, effectively, wrong. If events are simultaneous in the absolute rest frame, then they are presumably simultaneous in all reference frames, even if observers disagree on the time co-ordinates.


DaleSpam said:
No, it is not necessary to invent new terminology nor to change the definitions of existing terminology in order to understand a theory. You will be much better served to learn and use the existing terminology and definitions.

In addition, it is against the rules of the forum to do so unless you can produce a mainstream scientific reference supporting your term. Remember that this site is primarily educational in nature, and other people besides you will read what is written here. Your introduction of new terms and redefinition of existing terms will not help you learn and will cause unncessary confusion to others reading.
Again, apologies, the intention wasn't to invent new terminology, but to outline my own understanding.
 
  • #236
DaleSpam said:
mangaroosh, do you have any questions or confusions about SR itself that do not involve comparisons with or concepts from LR? If not, then your stated goals are accomplished and all you have to do is let go of LR like the rest of the scientific community did over a century ago.
I think it is difficult to eschew LR when discussing the necessary consequences of the Lorentz transform; given that LR also uses it, and is equally supported by experimental evidence, a comparison is necessary to identify what it necessarily implies and what it doesn't.

That would be my reasoning, anyway.
 
  • #237
mangaroosh said:
My understanding of this would be that "local clocks", therefore, cannot be used to determine the simultaneity of events. Only the absolute frame can determine if events are simultaneous or not. If events are simultaneous in the absolute rest frame, then they are simultaneous in all reference frames even if the observers disagree because of the time co-ordinates provided by their inaccurate clocks..

All this is accurate and good as long as you understand it is all predicated on an unproved and possibly perpetually unprovable assumption of the existence of an absolute rest frame. You are also apparently using the term simultaneous in a sense of meaning actually/absolutely simultaneous as opposed to the normal definition of equal clock readings ,,,,I hope you are now aware of the difference.

mangaroosh said:
If "local clocks" cannot determine "true time", and if their different times are the result of mechanics, then they presumably cannot be used to determine the simultaneity of events, in any meaningful way. .

It is true "local clocks" cannot determine "true time", but unfortunately we have NO clocks whatsoever that can tell true time . It is also true that clocks cannot determine simultaneity in the sense you seem to mean here . Even so they provide a "conventional simultaneity" which is quite meaningful in that it accurately correlates physical measurements with the mathematical structure we use to order and understand the world. And does it in all possible inertial frames.
Even if there is a preferred frame and all the clocks in other frames could be magically synched from them so there were all "actually" simultaneous , they would no longer work in all those other frames. Physical measurements would no longer be consistent but would vary from frame to frame and even with direction within any frame. To function we would still need conventionally synched clocks and RoS

mangaroosh said:
If events are simultaneous in the absolute rest frame, then presumably they must be simultaneous in all reference frames, even if the observers cannot determine this to be so, due to their inaccurate clocks. They might disagree on whether events are simultaneous or not, but presumably they can't really say; and instead of the Einsteinian notion that both reference frames are equal, it would be possible that, according to LET, that one observer actually is correct, because they might actually be in the absolute rest frame, just unable to determine it. .

Once again this all might possibly be accurate but is beyond determination
 
  • #238
GeorgeDishman said:
No, what it says is that all non-degenerate inertial coordinate systems are equally valid. Remember relativity is geometric so think of it in those terms: on a spacetime diagram the velocity of one object is the angle between their worldlines when they cross. What the PoR is saying is that it is meaningless to ask what the "angle between" is if you only have one line. In fact the same was true in the Newtonian world because it included Galilean Relativity but you can't use the geometric view.
That is essentially what the POR says, but the stated consequence of that is that observers cannot determine if they are in motion or at rest. It doesn't necessarily refer to the velocity of the object relative to something else, but to whether the object is in motion or at rest.

I'm not sure how beneficial it is to think of relativity as solely geometric; while it may be a geometric theory, it is a theory of the physical world, so we should be able to relate it back to the physical world. In that case, the stated consequence of the PoR is that if you try to conduct an experiment, wherever you are, to determine if you are moving or at rest, you will not be able to do so.


GeorgeDishman said:
Right, the context has been lost. Those will be important if you try to understand the Hole Argument regarding substantivalism, they are not relevant here but will be if you discuss it in the philosophy forum.
OK, cheers for that. I'll hopefully get to the point where I can move on to that.
 
  • #239
GeorgeDishman said:
True, often the intuitive view is misleading but in the case of SR it actually works quite well. What is necessary though is to replace the Newtonian view of time with something rather different. Once you get the hang of that, it all falls into place and is surprisingly elegant. I want to put together something that I can use in the future as this keeps coming up but it may take some time. If I can't do it soon, I'll try to give a verbal explanation here but give me a day or two yet please.
That sounds great. Take your time in doing it, and certainly don't rush on my account.

It might be worth clarifying that I (think I) have a decent enough understanding of the notion of relative time; the issue is when I subject it to critical reasoning I come to conclusions which are at odds with Einsteinian relativity; so there must be some issue in my reasoning or in my understanding of certain, key, concepts.


GeorgeDishman said:
In relativity, mass is invariant. It is one of those odd quirks of history that the effect known as "relativistic mass" as a physical change only happens in aether theory!
How does mass increase with speed though?


GeorgeDishman said:
Yes, that's a different ball game entirely. The block universe is a philosophical interpretation which only takes an input from relativity. There's a thread running on that in the philosophy group regarding A-series or B-series.
Cheers, I've posted in that thread, but must go back to it again.

Would it be fair to say that the block universe appears to be the logical conclusion of relativity?
 
  • #240
GeorgeDishman said:
OK, that has been the impression I was getting, my apologies if I got that wrong.
No need to apologise, I think it can be a natural dynamic of such discussions. When I understand the point I am making, I tend to presume that others will as well, so I might not always make the points clearly enough.


GeorgeDishman said:
Correct.

Correct.

In that case there is no problem the key is that while "relative" and "absolute" are different, both phrases refer to the same concept of simultaneity. That's what you said above so were are good up to here.
I think we're in agreement on here, but just to clarify. The term "simultaneity" in both concepts refers to the same thing i.e. happening at the same time.

But both are different concepts, in and of themselves.


GeorgeDishman said:
Is there or isn't there? You just said you weren't clear on it and then you say that it is clear there is no disagreement? This is very important because that is the single definition of simultaneity which the previous phrases share (and which we address later).
Again, apologies, I didn't elucidate that particular point clearly enough.

There is no disagreement over the concept of "simultaneity". I think we agree that it means "happen at the same time"

What I am not clear on is whether the equality of numerical values is a necessary requirement for determining simultaneity in Lorentzian relativity. As I see it, it boils down to a question of what the time co-ordinates represent.

In Lorentzian relativity "local clocks" don't tell the "true time", so I can't see how inferences can be made concerning the simultaneity of events, from inaccurate clocks. Those clocks can be used for the purpose of transforming co-ordinates from one reference frame to another, but I don't see how inferences about simultaneity can be made.

The inference of simultaneity of events appears to be only possible in the absolute rest frame, such that if they are simultaneous in that reference frame then they are simultaneous in all reference frames, regardless of the disagreement between observers.

That we cannot determine the absolute rest frame, is not necessarily important in determining whether LR incorporates AS, or not.


GeorgeDishman said:
Well you've got three of us all telling what the definitions are so that shouldn't be a problem.
And I think I have demonstrated that I have an understanding of those concepts; the issue lies in the conclusions I come to when applying critical reasoning to those concepts.



GeorgeDishman said:
It is the definition of simultaneity which is common to both the relative and absolute alternatives. I'll say it again slowly: the word "simultaneous" means "at the same time". "The same" in mathematically means "having equal values". The values assigned against a axis are called "coordinates" so "simultaneous" means "having the same time coordinate values".
...

Whether the times so allocated from local clocks are "equal for all observers if equal for one" or "equal for one but not for others in motion relative to the first" is the difference between absolute and relative simultaneity.
OK, the point I was making was slightly different, but I don't think it is necessary to go into it. The formulation above helps to clarify what I am trying to get at.

We agree on the idea that simultaneous means "happen at the same time"; where the issue is, as I see it, is in whether or not "at the same time" means having the same time co-ordinate values. The understanding I have is that under Einsteinian relativity it does mean this, but that it doesn't necessarily mean this under Lorentzian relativity.

The reasoning that has lead me to this conclusion is that the time co-ordinates provided by "local clocks", under the Lorentzian interpretation, are not the "true time" co-ordinates of an event. The "true time" co-ordinates can only be provided by a clock at rest in the absolute rest frame, and so, only these time co-ordinates can determine if events are actually simultaneous, or not.




GeorgeDishman said:
Local clocks are used in all the theories because otherwise you introduce an unknown component concerning the signal travel time between the clock and that which is being timed. Lorentzian philosophy included presentism but in the scientific theory clocks behaved identically to those in relativity.
This might be more helpful in clarifying the distinction, because I can't see how RoS is compatible with presentism i.e. the idea that there is only one, single present moment for all observers; the present moment isn't relative, it is the same for all observers.

This would mean that events which are simultaneous in the present of one observer, would have to be simultaneous in the present of all observers, since there is only one present moment for all observers. This would be the case despite the differing time co-ordinates provided by "local clocks" of relatively moving observers.


GeorgeDishman said:
No, generally we just talk of clocks and time, it is the method that matters and that's always the same. A time is what you read from a clock.
But the underlying assumptions about time are fundamentally different, such that what those clock readings represent are fundamentally different.


GeorgeDishman said:
Yes, and so are you :wink:. Look back at what you said at the top of your reply:
I'm not certain about the claim that Lorentz's model didn't exhibit absolute simultaneity; insofar as it incorporates presentism, then it presumably must incorporate AS; for the reason outlined above.


GeorgeDishman said:
The two statements are equivalent, agreed?
I might be misunderstanding the point you are making here, but I wouldn't say the statements are equivalent at all, I would say that the two statements are incompatible.

The concept of simultaneity is the same in both, but RoS refers to the joint possibility of simultaneity and non-simultaneity of events, while AS only allows for one or the other; such that RoS is not compatible with AS because AS doesn't allow for the conditions which would constitute RoS.

This is something you agreed to above, so I must be misinterpreting what you mean by equivalent.


GeorgeDishman said:
I don't think there's any need now. Having resolved that, I would suggest you lay LET aside for the moment, it's philosophy is diametrically opposed to what you need to learn if you are to understand SR and will make it very difficult for you. You can always come back to it once you grasp SR, that's what I did.
I think we're getting closer to resolving it, anyway. A couple of the points you made have helped to clarify a number of the issues.

Ultimately I want to understand the physical world better; this requires an understanding of SR, but is not limited to it. In the context of this discussion, however, a large part of the question can be boiled down to what the necessary consequences of the Lorentz transform are; in that context, understanding LET is important, as it also utilises the transform; if there are differences in the theories, then we can make certain deductions about what are and what aren't the necessary consequences of the transform.
 
  • #241
mangaroosh said:
My understanding of this would be that "local clocks", therefore, cannot be used to determine the simultaneity of events. Only the absolute frame can determine if events are simultaneous or not. If events are simultaneous in the absolute rest frame, then they are simultaneous in all reference frames even if the observers disagree because of the time co-ordinates provided by their inaccurate clocks.
You are conflating Galilean transformation and Lorentz transformation. Prior to Maxwell, the dominate view was the principle of relativity based on the Galilean transform in which time and space were separate and time did not enter into the transformation so that all frames had the same absolute time. But it wouldn't make sense to speak of an "absolute frame" or an "absolute rest frame". There was no preferred frame. The transformation operated only on one spatial dimension along the direction of motion.
mangaroosh said:
OK, but the current state of affairs is that LR uses the LT, no?
Yes, but remember, it was developed after SR and is nothing more than SR with a preferred frame which is what you get when you substitute Einstein's second postulate with the postulate that light propagates at c only in a single elusive absolute rest frame.
mangaroosh said:
If "local clocks" cannot determine "true time", and if their different times are the result of mechanics, then they presumably cannot be used to determine the simultaneity of events, in any meaningful way.
True.
mangaroosh said:
If events are simultaneous in the absolute rest frame, then presumably they must be simultaneous in all reference frames, even if the observers cannot determine this to be so, due to their inaccurate clocks.
Again, this is conflating concepts from LET and the Galilean PoR where all reference frames agree on the absolute time and where there is no need for an absolute rest frame and where there are no inaccurate clocks.
mangaroosh said:
They might disagree on whether events are simultaneous or not, but presumably they can't really say; and instead of the Einsteinian notion that both reference frames are equal, it would be possible that, according to LET, that one observer actually is correct, because they might actually be in the absolute rest frame, just unable to determine it.
Everybody and everything is in every reference frame. But if you mean that if an observer is at rest in the absolute rest frame, his clock would be ticking at the same rate as the coordinate clocks in the absolute rest frame tick, then that would be correct. But it would also be correct to say that in another frame moving at v in the x direction, if there were an observer moving at v in the -x direction, then his clock would be ticking at the same rate as the coordinate clocks in the absolute rest frame. When we're talking about simultaneity, we are not concerned about an observer's clock and its motions or rest state in a frame. We are talking about the infinite number of coordinate clocks that define time in a particular frame and whether or not two events at different locations occurred at the same time.
mangaroosh said:
It might be worth mentioning that, if time did not exist, a notion mentioned in this, mainstream source, then it would arguably be pretty much identical to the notion of absolute time, insofar as presentism would be prevalent. In that instance, the "relative time" of local clocks could not be used to determine RoS.

I don't think that is how LET is formulated presently though is it? Although I think the notion would be more compatible with LET than Einsteinian relativity.
No, that's not worth mentioning.

As I have said before, present day LET is usually exactly the same as SR but with a presumed preferred frame in which light propagates at c and in no others.
 
  • #242
I've rearranged the order of your response, but I don't think it misrepresents you - please let me know if you believe it does. I've taken your response to one point and made it the main focus of this post, because it cuts straight to the heart of the question; I've replied to the rest below, but this I think might offer the quickest route to reolution. This is followed by a point which is directly relevant to that.


mangaroosh said:
Does Lorentzian relativity incorporate AS?
harrylin said:
Yes, but perhaps not explicitly: he referred to it as "true time".
OK, this is the critical issue, as I see it.

It might be helpful to re-state the definitions again, for clarity:

Absolute simultaneity means that events which are simultaneous in one reference frame, are simultaneous in all reference frames.

RoS is where events that are simultaneous in one reference frame are not, necessarily simultaneous across all reference frames; that is, it allows for the joint possibility of simultaneity and non-simultaneity across reference frames.

AS doesn't allow for the joint possibility of simultaneity and non-simultaneity across reference frames; therefore, AS and RoS are incompatible.


If LR incorporates AS, then it can't incorporate RoS; if LR utilises the Lorentz transform, then RoS cannot be a necessary consequence of the LT.


Under Lorentzian relativity, if events are simultaneous in the absolute rest frame, and that frame only, then they are simultaneous across all reference frames - irrespective of differing time co-ordinates from "local clocks".

harrylin said:
"Absolute simultaneity" commonly refers to the same unique reference frame as "absolute velocity". See aslo:
http://en.wikipedia.org/wiki/Absolute_time_and_space
I hope that you see that that resolves all confusion on this matter. :smile:
I understand that AS is linked to the same unique reference frame as absolute velocity, but I don't think alluding to it sufficiently explains the "absolute velocity" and "velocity is absolute" point being made.

As above, under Lorentzian relativity, if events are simultaneous in the absolute rest frame, and that frame only, then they are simultaneous across all reference frames - irrespective of differing time co-ordinates from "local clocks".


harrylin said:
As Lorentz's concise statement showed, it is mistake to think of "in contrast".
Instead, simply put: SR + ether = LR.
Similarly: SR + block universe = MR.
We don't necessarily need to think of it in contrast, we can think of it in conjunction with, but there are differences in the models which automatically lend themselves to contrast, or comparison.


If absolute simultaneity means, events that are simultaneous in one reference frame i.e. the absolute reference frame, are simultaneous in all reference frames, then it means that simultaneity (of those events) is absolute.


harrylin said:
I'm quite sure that I let you read the explanation from the original paper in a discussion here not long ago... Now I have no time to search it back again but in a nutshell, it was already applied before time dilation was added to the transformations of Lorentz.
OK, I can't specifically remember it in this, or any similar context, but I may just not have made the connection. It isn't really an essential point I don't think, so I won't labour it.

harrylin said:
Yes - exactly! Didn't you read it? :uhh: If you had searched for it with "simultaneous" you would have found it in a minute... and the first pages of that article are much shorter than the total of pages that you had us write here in discussions with you...
As a matter of fact, if you don't paste that passage in your next reply to me, I will not reply to you for at least one month. :grumpy:
I had already read it before you posted it; I was more trying to re-iterate that the concept of RoS had been understood.

That, along with the subsequent questions, were intended to show the path of logic that I was following, to see if there was an issue there. The answer to each individual question would have helped determine where I am going wrong. I was confident that it wasn't on the point of RoS, but just stated it more as a "road marker".

Is this the relevant passage
Definition of Simultaneity
We have to take into account that all our judgments in which time plays a part are always judgments of simultaneous events. If, for instance, I say, “That train arrives here at 7 o'clock,” I mean something like this: “The pointing of the small hand of my watch to 7 and the arrival of the train are simultaneous events.”3
...
It is essential to have time defined by means of stationary clocks in the stationary system, and the time now defined being appropriate to the stationary system we call it “the time of the stationary system.”


harrylin said:
No difference after 1905 (before 1905 Lorentz did not have a clear idea about it, but Poincare already applied it to the time measured by clocks).
There is no difference in the readings, or behaviour of the clocks, but there is a difference in the underlying assumptions of what the readings, and behaviour represent, isn't there?


harrylin said:
Definitely not: observers can only use "local time", since they cannot know true time. :tongue2:
But this would just mean that they can't know the true time co-ordinates for events, as opposed to implying that the events are not simultaneous. They would disagree about simultaneity, but unlike the Einsteinian interpretation, there is the possibility that one, or both of them is wrong; because one or both of their clocks doesn't tell the "true time".

Presumably it is the "true time" co-ordinates of events that determines whether or not they are simultaneous, as opposed to the inaccurate, or "not-the-true-time", co-ordinates.


harrylin said:
1. That has nothing to do with Einstein's train illustration which you claimed to understand and which you claimed to comment on - thus your claim was totally wrong. So, please explain Einstein's train example, as a test. :devil:
We may be arguing over semantics here.

I'll speak in terms of Albert and Henry moving relative to each other, as per the video posted in this, or another thread; from Albert's perspective, Henry's clock runs slower because the photon in his clock has to travel a longer distance between mirrors i.e. the path represented by the hypotenuse of a right angled triangle.

If, however, the speed of light is c for all observers, it means that lengths must contract and time must slow down for Henry (according to Albert). This means that, according to Albert, Henry's clock ticks slower.

The phrase I would use here is to say that Henry's clock ticks at a slower rate.


harrylin said:
2. Your logic appears very incomplete to me - but that's irrelevant at this point.
That is the primary motivation for posting here, to find where the error in the logic is and to, hopefully, correct it.


harrylin said:
Here's again the key section that you can't have missed (but of which you did read the context so that you surely won't misunderstand it) - and funny enough, it appears that the first section wasn't authored by Lorentz:

(1)"it was necessary incidentally to throw over the one universal time, and substitute local times attached to moving bodies and varying according to their motion. The equations on which the theory of relativity is based are due to Lorentz, but Einstein connected them with his general principle, namely, that there must be nothing, in observable phenomena, which could be attributed to absolute motion of the observer. [..] In orthodox Newtonian dynamics the principle of relativity had a simpler form, which did not require the substitution of local time for general time. " [..]
(2)"It is not necessary to give up entirely even the ether. [..] In my opinion it is not timpossible that in the future this road, indeed abandoned at present, will once more be followed with good results, if only because it can lead to the thinking out of new experimental tests. Einstein's theory need not keep us from so doing; only the ideas about the ether must accord with it."

Now, the "time" of a system in rest with the ether is often called "absolute" or "true" time, consistent with Newton's defnitions. I hope that it is now clear to you that such a "true time" is not at all incompatible with RoS, just as also our "universal time" is not incompatible with RoS - even if astronauts use it. As a matter of fact, astronauts could use both local and universal time, and thus use "dual time" - that would be a neat example of RoS. :rolleyes:
I'm not 100% sure of the intention of the initial part of the post; the concept of RoS under Einsteinian relativity isn't the issue as such. The question pertains to the notion of RoS under Lorentzian relativity, so that we can deduce what the necessary consequences of the Lorentz transform are.

While Einsteinian relativity abandoned the idea of one universal time, Lorentzian relativity retained it; and while both theories utilise the concept of "local clocks", the underlying assumptions as to what they represent are materially different; to the extent that the Einsteinian interpretation leads to the notion that time is relative, while Lorentzian relativity retains the notion of universal, or "true time". This means there are fundamentally different assumptions about time and clocks, and therefore the time co-ordinates of events.


It seems like you are suggesting that because ER uses "local clocks" that AS and RoS are compatible; but as mentioned, there are fundamental, underlying assumptions, about what the time co-ordinates of those clocks represent, which pertains directly to the simultaneity of events.

If I might be so bold as to ask you to spell out how the notion of absolute time is not inconsistent with RoS, because you seem to stop just short of it every time; it could simply be that I am not making the connection, but I can't seem to. I have the tendency to assume that the points I make are going to be understood also, but I often find that spelling it out clearly allows for any issues to be clearly identified.


As mentioned, my understanding would be that the simultaneity of events is determined by the "true time" co-ordinates of events, as opposed to the "untrue" time co-orindates.


harrylin said:
Good! :smile:
Should get time this evening; I was busy replying to posts when I got the chance during the day.


harrylin said:
Yes indeed! And as I mentioned before, a lot of expressions had acquired multiple compatible meanings before SR, which became incompatible afterwards. Consequently, one often has to consider the context to understand what a writer means when using certain words.
OK, but is there a conceptualisation of AS other than the one I have been referring to?
 
  • #243
Austin0 said:
All this is accurate and good as long as you understand it is all predicated on an unproved and possibly perpetually unprovable assumption of the existence of an absolute rest frame. You are also apparently using the term simultaneous in a sense of meaning actually/absolutely simultaneous as opposed to the normal definition of equal clock readings ,,,,I hope you are now aware of the difference.
I do understand that it is predicated on said assumption, but that is, in itself, a separate matter. The point of contention is whether or not RoS is a necessary consequence of the Lorentz transform. If my understanding is accurate, as you say, then the conclusion that can be drawn is that RoS is not a necessary consequence of the Lorentz transform; insofar as RoS is the joint possibility of events being simultaneous and non-simultaneous, across reference frames.

The point about equal clock readings is mentioned in the part that you said was accurate. It suggests that equal clock readings, across "local clocks", is part of the definition of simultaneity under the Einsteinian interpretation, but not necessarily the Lorentzian. Under the Lorentzian interpretation, it seems as though "local clocks" can disagree on the times of events, but the events would still be simultaneous, if they have the same time co-ordinates in the absolute rest frame only.

That is, if the "true time" co-ordinates of events are the same, then they are said to be simultaneous, under Lorentzian relativity; if, however, the time co-ordinates of local clocks, in different reference frames, are the same, then events are not necessarily simultaneous - or perhaps definitely not simultaneous - because those co-ordinates don't represent the "true time" co-ordinates of the events.


Austin0 said:
It is true "local clocks" cannot determine "true time", but unfortunately we have NO clocks whatsoever that can tell true time . It is also true that clocks cannot determine simultaneity in the sense you seem to mean here . Even so they provide a "conventional simultaneity" which is quite meaningful in that it accurately correlates physical measurements with the mathematical structure we use to order and understand the world. And does it in all possible inertial frames.
Even if there is a preferred frame and all the clocks in other frames could be magically synched from them so there were all "actually" simultaneous , they would no longer work in all those other frames. Physical measurements would no longer be consistent but would vary from frame to frame and even with direction within any frame. To function we would still need conventionally synched clocks and RoS
We can't necessarily say that we have no clocks that can tell the "true time", because we cannot determine the motion or rest of any reference frame. That, however, is again a separate matter to the question of whether or not RoS is a necessary consequence of the Lorentz transform.

The point you raise, however, is pertinent to a thread that was locked in the General Discussion forum, so unfortunately I don't think we can go into it here.


Austin0 said:
Once again this all might possibly be accurate but is beyond determination
There is an alternative conventionw which could help determine it, but again, it is the subject of a locked thread.

Again, however, it is a separate point to the question of whether or not RoS is a consequence of the Lorentz transform.

If you are correct, and the point above is accurate, then the logical deduction would be that it isn't necessarily a consequence of the Lorentz transform, but an interpretation of it.
 
  • #244
mangaroosh said:
I tend to find that I develop a better understanding of something when I subject it to critical reasoning; the distinction between actual and measured values came about on the basis of that reasoning. It also seems to be an important distinction, again for the reasoning that was outlined. Again, based on logical reasoning, it would seem that RoS is a consequence of the actual speed of light remaining invariant, but the actual speed of light would be unmeasurable, given the distinction between actual and measured speeds.

As much as I would like to eschew reason in my attempts to understand relativity, it isn't something that comes naturally.
Nonsense. Belief in and obsession over undetectable entities is the purview of ghost stories and superstition, not reason. You do not need to "eschew reason" in order to understand SR.

Btw, the language of reason is math, so if you really want to use reason then you need to learn the math. So far, I have seen no indication of that from you. In fact, I think that you do not want nor embrace reason, but rather intuition. I think your trouble with SR is that it is not intuitive; you are having trouble eschewing intuition and embracing reason.

mangaroosh said:
This is where the point of contention lies. According to LR, only the absolute rest frame has the "true time", so presumably only this frame can be used to determine the simultaneity of events.
This is a different concept from the AS concept presented above, and it is much more in line with the language of LET. Let's look at these two concepts.

AS, as you defined it above, requires a set of at least two equivalent reference frames and a transformation between them. If the transformation between the frames is such that any pair of events which have the same time coordinate in one frame have the same time coordinate in all of the equivalent frames then there is AS. If not, there is RoS. This concept of AS is opposite to RoS, and can be determined simply by looking at the transformation between the equivalent frames.

LET's concept of "true time" identifies a single reference frame, the aether frame, as being unique and not equivalent to any other frame. Time in the undetectable aether frame is called "true time", and so events which are simultaneous in the aether frame would be truly simultaneous. There is no reference to nor comparison of time in different frames, so this is not AS as defined by you above. I don't know if Lorentz ever used the term, but based on the terminology that he did use he would probably call it "true simultaneity".

Note that true simultaneity (refers to simultaneity in a single frame) is not the opposite of relativity of simultaneity (compares simultaneity in multiple frames). LET has RoS, it also has this concept of true simultaneity which is distinct from your and my concept of AS.

mangaroosh said:
If events are simultaneous in the absolute rest frame, then they are presumably simultaneous across all reference frames
This is incorrect. Simply apply the definition of simultaneity to determine if two events are simultaneous or not. That events are simultaneous in the aether frame does not imply that they are simultaneous in all frames. In fact, the LT from the aether frame to the local frames guarantees that is not the case.

mangaroosh said:
Surely where two theories are experimentally equivalent to one another theory, both theories must be someway understood before one can be rejected in favour of the other? How else could it be determined that they are experimentally equivalent?
This is true, but not relevant to your stated goal of learning SR. You do not need to reject LET in favor of SR in order to learn SR. Nor do you need to determine that they are experimentally equivalent in order to learn SR.

mangaroosh said:
If the time co-ordinates differ then, according to LR, it is because the "local clocks" are, effectively, wrong.
Hence the schizophrenic nature of LET: It purports to be a scientific theory, science uses experimental measurements to test theories, but measurements are wrong according to the theory.
 
  • #245
ghwellsjr said:
You are conflating Galilean transformation and Lorentz transformation. Prior to Maxwell, the dominate view was the principle of relativity based on the Galilean transform in which time and space were separate and time did not enter into the transformation so that all frames had the same absolute time. But it wouldn't make sense to speak of an "absolute frame" or an "absolute rest frame". There was no preferred frame. The transformation operated only on one spatial dimension along the direction of motion.
I don't think I am conflating the two, am I?

The concept of reference frames is relevant to Galielan relativity as well isn't it? Galileo's observer on the ship is a particular example, no?

The Galilean Principle of Invariance, or the consequence of it, suggests that an observer cannot determine if they, and by extension their reference frame, are in motion or at rest. This referred to absolute motion and absolute rest, with the implication that they were in one or the other state. If they were indeed at absolute rest, then presumably the concept of an absolute rest frame would make sense.

Is Lorentzian relativity not based on the notion of absolute space, or does it not incorporate it? My understanding was that the Lorentz transform determines the physical contractions of objects as they moved through the ether, or relative to the absolute rest frame.


ghwellsjr said:
Yes, but remember, it was developed after SR and is nothing more than SR with a preferred frame which is what you get when you substitute Einstein's second postulate with the postulate that light propagates at c only in a single elusive absolute rest frame.
Would it be accurate to say that it is SR with absolute time and space, as opposed to relativised spacetime; where the contractions of objects are physical, as opposed to geometrical (is that correct??), and are due to the motion relative to the absolute space; and the Lorentz transform allows observers to determine to what extent the physical instruments of a relatively moving reference frame are contracted, relative to their own reference frame?

ghwellsjr said:
True.
Does that not then mean, that even if events have the same time co-ordinates, after the transform, that it cannot necessarily be said that they are simultaneous, because at least one of those time co-ordinates is not the true time co-orindates of the events?

Equally, if events don't have the same time co-ordinates, after the transform, it cannot necessarily be said that they are not simultaneous, because, again, at least one of the time co-ordinates is not the true time co-ordinates of events?

ghwellsjr said:
Again, this is conflating concepts from LET and the Galilean PoR where all reference frames agree on the absolute time and where there is no need for an absolute rest frame and where there are no inaccurate clocks.
Would it be fair to say that LR is effectively just Gailean relativity with physical contractions?

ghwellsjr said:
Everybody and everything is in every reference frame. But if you mean that if an observer is at rest in the absolute rest frame, his clock would be ticking at the same rate as the coordinate clocks in the absolute rest frame tick, then that would be correct. But it would also be correct to say that in another frame moving at v in the x direction, if there were an observer moving at v in the -x direction, then his clock would be ticking at the same rate as the coordinate clocks in the absolute rest frame. When we're talking about simultaneity, we are not concerned about an observer's clock and its motions or rest state in a frame. We are talking about the infinite number of coordinate clocks that define time in a particular frame and whether or not two events at different locations occurred at the same time.
Apologies, the emboldened was indeed what I meant.

The reason the [traveling v in the -X direction] observer's clocks would tick at the same rate as the absolue rest frame clocks, is because he would be in the absolute rest frame.

When talking about simultaneity, it has repeatedly been mentioned that events are simultaneous if they have the same time co-orindates; if time co-ordinates are provided by clocks, and the state of relative motion affects the time co-ordinates given by a clock, it is difficult to see how that isn't a factor.

The definition of simultaneity - whether or not two events at different locations occurred at the same time - is agreed upon. My reasoning would lead me to deduce that the underlying assumptions about the nature of time play a fairly pivotal role in determining the simultaneity of events, insofar as those assumptions affect the interpretation of what time co-ordinates represent.

Einsteinian relativity seems to suggest that events with the same time co-ordinates, provided by local clocks, are simultaneous; while events with different time co-ordinates are not simultaneous; because those time co-ordinates represent "proper time". Under ER it is possible that events are simultaneous across some reference frames, while the same events are not simultaneous across others.

It would seem that, under Lorentzian relativity, the time co-ordinates provided by local clocks (excluding the absolute rest frame clocks) cannot be used to determine the simultaneity of events because they do not represent the "true" time co-ordinates of events. It, also, doesn't seem to allow for the possibility that events are simultaneous across some reference frames, but not simultaneous across others; that is, if they are simultaneous in the absolute reference frame, then they are simultaneous in every reference frame because time is absolute, not relative.

ghwellsjr said:
No, that's not worth mentioning.

As I have said before, present day LET is usually exactly the same as SR but with a presumed preferred frame in which light propagates at c and in no others.
In the context of the question of whether or not RoS is a consequence of the Lorentz transform, you may be right, it probably isn't worth mentioning.

In a broader discussion on the concept of simultaneity, however, I think it is pretty relevant.

LET does incorporate absolute space and time though, doesn't it?


Just as an aside, would you now of any good resources on LET, or neo-Lorentzian relativity? Not necessarily for understanding SR better, but just for the purpose of developing an all round understanding of scientific theories.
 

Similar threads

  • Special and General Relativity
2
Replies
36
Views
1K
  • Special and General Relativity
2
Replies
54
Views
697
  • Special and General Relativity
Replies
17
Views
571
  • Special and General Relativity
Replies
20
Views
801
  • Special and General Relativity
4
Replies
116
Views
6K
  • Special and General Relativity
3
Replies
89
Views
6K
  • Special and General Relativity
Replies
27
Views
1K
  • Special and General Relativity
Replies
29
Views
1K
  • Special and General Relativity
Replies
16
Views
1K
  • Special and General Relativity
Replies
21
Views
1K
Back
Top